- PowerScore Staff
- Posts: 5972
- Joined: Mar 25, 2011
- Tue Jul 19, 2016 4:13 pm
#27316
Complete Question Explanation
The correct answer choice is (C)
With the last rule suspended, the setup appears as follows:
This setup provides sufficient information to attack the questions.
Answer choice (A) is incorrect because from the third rule either S or T must be included in the main-dish recipe.
Answer choice (B) is incorrect because from the second rule either F or N must be included in the main-dish recipe.
Answer choice (C) is the correct answer choice.
Answer choice (D) is incorrect because from the third rule both S and T cannot be included in the main-dish recipe.
Answer choice (E) is incorrect because from the second rule both F and N cannot be included in the main-dish recipe.
The correct answer choice is (C)
With the last rule suspended, the setup appears as follows:
This setup provides sufficient information to attack the questions.
Answer choice (A) is incorrect because from the third rule either S or T must be included in the main-dish recipe.
Answer choice (B) is incorrect because from the second rule either F or N must be included in the main-dish recipe.
Answer choice (C) is the correct answer choice.
Answer choice (D) is incorrect because from the third rule both S and T cannot be included in the main-dish recipe.
Answer choice (E) is incorrect because from the second rule both F and N cannot be included in the main-dish recipe.
You do not have the required permissions to view the files attached to this post.
Dave Killoran
PowerScore Test Preparation
Follow me on X/Twitter at http://twitter.com/DaveKilloran
My LSAT Articles: http://blog.powerscore.com/lsat/author/dave-killoran
PowerScore Podcast: http://www.powerscore.com/lsat/podcast/
PowerScore Test Preparation
Follow me on X/Twitter at http://twitter.com/DaveKilloran
My LSAT Articles: http://blog.powerscore.com/lsat/author/dave-killoran
PowerScore Podcast: http://www.powerscore.com/lsat/podcast/